Page 1 of 1

Again PHP

Posted: Mon Feb 22, 2021 10:47 am
by Asif Hossain
In a $n \times n$ grid show that if you choose any $2n-1$ points at least $3$ points would make a right triangle.
Source: May be bdmo (actually it said it bdmo 2012 secondary 8 but when i checked it was not :?: )

Re: Again PHP

Posted: Thu Feb 25, 2021 3:57 pm
by Asif Hossain
Update: i have got close to proving it through induction but the arguments are not as pretty as pigeonhole principal...
(it included deleting horizontal and vertical lines but no pretty PHP :x :x )

Re: Again PHP

Posted: Mon Mar 08, 2021 2:52 am
by Anindya Biswas
Asif Hossain wrote:
Thu Feb 25, 2021 3:57 pm
Update: i have got close to proving it through induction but the arguments are not as pretty as pigeonhole principal...
(it included deleting horizontal and vertical lines but no pretty PHP :x :x )
In that case feel free to share your solution here.

Re: Again PHP

Posted: Mon Mar 08, 2021 11:30 am
by Asif Hossain
Anindya Biswas wrote:
Mon Mar 08, 2021 2:52 am
Asif Hossain wrote:
Thu Feb 25, 2021 3:57 pm
Update: i have got close to proving it through induction but the arguments are not as pretty as pigeonhole principal...
(it included deleting horizontal and vertical lines but no pretty PHP :x :x )
In that case feel free to share your solution here.
It is not complete and i also couldn't write it formally. But the main idea was add a extra row and column and you have to at least 3 points from the extra portion due to induction hypothesis then you have to delete some columns to avoid right triangle. then i stucked..(ik that it must be that the smaller rectangular portions must have a right triangle but i couldn't prove it...)(i doubt that it can be solved by some pretty php but i couldn't have insight on it so i gave up :cry: )

Re: Again PHP

Posted: Thu Mar 11, 2021 1:00 am
by Zafar
well how can it satisfy n=2 ? I think we can select exactly n+1 points maximum that wont make a right triangle . hence for all n> 2 , 2n-1>n+1 . and so I think its done .

Re: Again PHP

Posted: Fri Mar 12, 2021 7:23 pm
by Anindya Biswas
Zafar wrote:
Thu Mar 11, 2021 1:00 am
well how can it satisfy n=2 ? I think we can select exactly n+1 points maximum that wont make a right triangle . hence for all n> 2 , 2n-1>n+1 . and so I think its done .
It is possible to choose $n+1$ points without making a triangle :roll: